PT21.S3.Q08 - "Smart highway" system [Posting on behalf of a 7Sage user]

Juliet - Student ServiceJuliet - Student Service Member Administrator Student Services
edited July 2021 in Logical Reasoning 5740 karma

[I am posting on behalf of a 7Sage user. Please feel free to leave your comments below. Thank you for your help!]

I did not understand this question. Can anyone please offer a detailed explanation of this one that can help me further attack future questions regarding this question type?

https://7sage.com/lsat_explanations/lsat-21-section-3-question-08/

Comments

  • aman.sanghavi7aman.sanghavi7 Alum Member
    edited July 2021 41 karma

    stimulus
    Eva: we should incorporate a "smart highway" system to help drivers take alternate roads and avoid congestion. This system would reduce traffic congestion and an added bonus is a decrease in the loss of money and productivity that is a result of this congestion.

    Luis: We don't really need this, we already have radios that provide drivers with traffic reports.

    AC

    A: if we implement this system, the cars won't breakdown anymore. this is providing an AC that is outlandish. just because a highway system changes doesn't mean your car's suspension system will last forever.

    B: mentions traffic lights which are not mentioned anywhere in the passage or even referred to.

    C: makes the assumption that the traffic congestion mentioned can be improved. if the congestion is way past the point where a change in highway systems wouldn't make a difference, Eva's argument falls apart.

    D : who cares about different systems in different cities. that doesn't matter.

    E: weakens the argument because it states that any old vehicle would not be able to integrate with/ benefit from this smart system.

  • DarinnnnnDarinnnnn Member
    244 karma

    how I looked at it was that the conclusion has a "should" in it which means the author is saying we should do something and give reasons that benefit of us doing it. But what if we can't do it? The author fails to consider that our highway system may be so bad that we can't improve it or that we don't have the funds for it at all. The author isn't giving the full picture. For conclusions that end in should, we need to build a bridge to make sure that what we should do is actually feasible and that we actually want to do it.

  • edited July 2021 179 karma

    Hi there,

    When looking at necessary assumptions, go to the granular and ask yourself how do I make sure this (conclusion) happens? Ask yourself with answer choice "if conclusion true, then must this be true?"

    The granular in this case is that SHS can be installed. Should is recommendation language, you would not recommend something if it cannot happen. If it was the case that there is no need for SHS or SHS is inaccessible or corrupt, then SHS cannot happen. Why would you recommend something that cannot happen?

    Side track discussion: say you are given a conclusion that "Coach ate peanuts at the game". How do you make sure that this happens, at the granular level? My guesses would be coach is able to eat peanuts, peanuts were not inaccessible to the coach, peanuts were not bad, etc.

    a. must it be true that "SHS = no breakdown of vehicles that now cause traffic"? Maybe, how can I predict the future? Also, breakdown of vehicles is not in scope.
    b. must it be true that "traffic lights, coordinated = assure free flow of traffic". Once again, maybe. How can I know this? The stimulus only talks about traffic signs, which is what I think LSAT wants you to confuse. Signs do not equal lights.
    c. must it be true that"traffic flow not so bad that sig improvement impossible". Aha! So, SHS can be installed. If sig improvement (SHS) is impossible then how can you recommend installation of SHS?
    d. must it be true that "type of equipment would vary amongst cities". How is this relevant and in scope of the argument? Equipment is not something you can jump to. It is possible but it is not "must be true".
    e. must it be true that "older vehicles cannot be fitted to receive SHS signals"? Hmm, so there is a subset of vehicles where SHS is not advantageous. How does this make sure that my conclusion happens? It does not, it says the opposite. This would be okay for weakening but not for NA, where you are to say that the conclusion holds.

Sign In or Register to comment.